LSAT and Law School Admissions Forum

Get expert LSAT preparation and law school admissions advice from PowerScore Test Preparation.

User avatar
 Dave Killoran
PowerScore Staff
  • PowerScore Staff
  • Posts: 5852
  • Joined: Mar 25, 2011
|
#26969
Complete Question Explanation
(The complete setup for this game can be found here: lsat/viewtopic.php?t=11303)

The correct answer choice is (B)


If Otto does not participate in R during the year, then Otto must participate in M during the fall (and Nikki must participate in R during the fall). Because Otto does not participate in R and he participates in M during the fall, the only available sport for Otto in the spring is K. With Otto’s sports fully determined, Nikki’s sports can also be determined. We have already inferred that she must participate in R during the fall, and we determined during the setup that she must participate in V during the winter. For spring, she can no longer participate in R (because she participates in R during the fall), and because Otto participates in K during the spring she can no longer participate in K. Thus, in the spring Nikki must participate in M. Accordingly, in the summer her only option is K, and all of the season sports have been determined:
Feb 95_game #4_M12_L4_explanations_game#5_23_diagram_1.png
In this Cannot Be True question, answer choice (B) is thus correct.
You do not have the required permissions to view the files attached to this post.

Get the most out of your LSAT Prep Plus subscription.

Analyze and track your performance with our Testing and Analytics Package.